TL 2 chapter 13 #46

Collapse
X
 
  • Time
  • Show
Clear All
new posts
  • William Barron
    Junior Member
    • Nov 2010
    • 2

    TL 2 chapter 13 #46

    I have to admit that once again I am confused with how the answer key and the principle line up. Writing about Tertullian's criticism of Trajan and Marcus Aurelius it appears that your terms are anything but clear. The "terms" "no restrictions on hunting them down" and "should not punish them when they are brought to your attention" appear to be somewhat similar terms but one with a "not" in front of it. Then, in the conclusion you introduce an apparently completely unrelated term, "Your policy is unjust."

    This example does not appear to fit any of the forms given in the text. The best I can come up with is

    If P then Q; If not P then not Q
    Either P or not P
    ? (R)

    OR

    If P then Q; If R then S
    Either P or S
    Either Q or S (but a new term is introduced instead)

    I really do want to understand this but quite frankly it is frustrating when the examples in the homework do not follow even closely the examples given in the text. Then, try to teach it to students. Forget it. I would really appreciate ANY help you can add to this particular situation and in general why the principles given in the text are not always followed through in the homework problems.

    William
  • spiland
    Director, MPOA
    • Jun 2008
    • 454

    #2
    William,

    I'm not sure why it read the way that it does, because it does not seem to me to actually be Tertullian's argument, historically speaking. The dilemma should read something something like this:

    If Christians have committed crimes then Trajan's/Aurelius' policy is unjust (because they don't hunt down Christians), AND, If Christians have not committed crimes then Trajan's/Aurelius' policy is unjust (because they are punishing Christians simply because of religion).

    Either Christians have committed crimes or they have not.

    Therefore, Trajan's/Aurelius' policy is unjust.


    That is how it should read, William, and I can definitely see how you are confused by this, since this argument is totally different from the one given in your book, with respect to the consequent of the horns. I will get it marked in our proof copy to make sure it gets changed the next time.

    Comment

    Working...